Conformal Mapping: Finding \phi(z) = z^{0.5}

  • Thread starter Thread starter Niles
  • Start date Start date
Click For Summary
SUMMARY

The discussion focuses on the rigorous approach to finding conformal mappings, specifically for the function \(\phi(z) = z^{0.5}\). The mapping of the domain defined by \(r\exp(i\phi)\) (where \(r > 0\) and \(0 \leq \phi \leq \pi\)) is analyzed. The key transformation is \(\phi(re^{i\theta}) = r^{1/2}e^{i\theta/2}\), which leads to determining the possible values of \(r^{1/2}\) and \(\theta/2\) to identify the resulting region in the complex plane.

PREREQUISITES
  • Understanding of complex functions and their properties
  • Familiarity with conformal mappings and their applications
  • Knowledge of the exponential form of complex numbers
  • Basic skills in manipulating complex variables
NEXT STEPS
  • Study the properties of conformal mappings in complex analysis
  • Learn about the implications of the Riemann mapping theorem
  • Explore the geometric interpretation of complex functions
  • Investigate the behavior of branch cuts in complex functions
USEFUL FOR

Students of complex analysis, mathematicians focusing on conformal mappings, and anyone interested in the geometric interpretation of complex functions.

Niles
Messages
1,834
Reaction score
0

Homework Statement


Hi

Is there a rigorous way to find conformal mappins? Say I would like to find how \phi(z)=z^{0.5} maps the domain r\exp(i\phi) (with r>0 and 0\leq \phi \leq \pi), how would I do this?

Thanks in advance.
 
Physics news on Phys.org
Niles said:

Homework Statement


Hi

Is there a rigorous way to find conformal mappins? Say I would like to find how \phi(z)=z^{0.5} maps the domain r\exp(i\phi) (with r>0 and 0\leq \phi \leq \pi), how would I do this?

Please don't use the same symbol for two different objects in the same context! Either \phi is a complex function or it's the argument of a complex number. Choose one and stick with it, and find a different symbol for the other.

To answer your question: Start with \phi(re^{i\theta}) = r^{1/2}e^{i\theta/2}. What values can r^{1/2} take if r > 0? What values can \theta/2 take if 0 \leq \theta \leq \pi? What region of the complex plane does that give you?
 
Question: A clock's minute hand has length 4 and its hour hand has length 3. What is the distance between the tips at the moment when it is increasing most rapidly?(Putnam Exam Question) Answer: Making assumption that both the hands moves at constant angular velocities, the answer is ## \sqrt{7} .## But don't you think this assumption is somewhat doubtful and wrong?

Similar threads

Replies
4
Views
2K
  • · Replies 6 ·
Replies
6
Views
2K
  • · Replies 2 ·
Replies
2
Views
2K
Replies
6
Views
2K
  • · Replies 1 ·
Replies
1
Views
2K
  • · Replies 6 ·
Replies
6
Views
2K
  • · Replies 2 ·
Replies
2
Views
2K
  • · Replies 5 ·
Replies
5
Views
1K
  • · Replies 1 ·
Replies
1
Views
2K
  • · Replies 14 ·
Replies
14
Views
2K